LSAT and Law School Admissions Forum

Get expert LSAT preparation and law school admissions advice from PowerScore Test Preparation.

 Administrator
PowerScore Staff
  • PowerScore Staff
  • Posts: 8950
  • Joined: Feb 02, 2011
|
#27280
Complete Question Explanation

Weaken—#%. The correct answer choice is (C)

The vulnerability in this argument is that the conclusion—support groups do not help patients treated for disease T live longer, since 41/43 from both groups were dead after 10 years—still allows for the possibility that the support group patients did in fact live longer. Just because the same number from both groups were dead after 10 years does not mean the patients from both groups all died at the same time. For instance, what if the 41 dead support group patients all lived 9 years after the treatment, while the 41 dead patients who did not attend support groups only lived for 1 year after the treatment? 41 from each group would still be dead in 10 years, but the support group patients would have lived considerably longer nonetheless.

Answer choice (A): While the argument could be made that this answer choice weakens the argument slightly, the 41/43 from both groups who died is much more persuasive evidence than the comparison of the 2 survivors from each group. Answer choice (C) is a much stronger answer in terms of the doubt it casts on the argument’s conclusion (in fact, it completely destroys the conclusion in the stimulus).

Answer choice (B): This argument is only about support groups related to the treatment of disease T.

Answer choice (C): This is the correct answer choice. By stating that the support group members lived an average of 2 years longer than patients who did not attend support groups, this answer severely weakens the conclusion in the stimulus.

Answer choice (D): This argument is based on numbers and data, so the faith (or lack thereof) of support group members is not applicable to the conclusion. To weaken you must find an answer that challenges the data given.

Answer choice (E): Again, the argument is about whether support group members live longer, not about how well the two groups coped with the disease.
 niketown3000
  • Posts: 16
  • Joined: Jan 13, 2012
|
#3707
Hello,

9-5, # 8: Both A & C are weaken? I thought there was only one AC that always did the task (not a question of which did it better)? why is A worse? is there any pattern for handling ACs that both do task, but you are trying to choose the better one? How often are more than one correct technically?
User avatar
 Dave Killoran
PowerScore Staff
  • PowerScore Staff
  • Posts: 5972
  • Joined: Mar 25, 2011
|
#3713
Hey Nike,

Answer choice (A) is an attractive answer choice, but I wouldn't agree that it weakens the argument. Let's look at the two answer choices and see how that works.

With answer choice (C), it's clear how the answer weakens the argument: even though equal numbers died, the support group members lived on average two years longer.

Answer choice (A) looks rather similar to (C), so I understand your thought that it too weakens the argument. But, unlike (C), answer choice (A) addresses a small subset of patients--the four who lived longer than 10 years. While this small subset could suggest that support groups help, it doesn't have to, in part because 2 people in each group is such a small sample. For example, what about the other 39 support group patients who didn't live longer than 10 years? What if they lived a shorter amount of time than their non-support comparison group. That would suggest that perhaps support groups don't help, and may in fact hurt (thus strengthening the argument, not weakening it). Thus, we can't really draw a conclusion about whether (A) weakens the argument.

Note that in comparing (A) and (C), answer choice (C) makes a statement about the group as a whole, whereas answer choice only addresses a small portion of the group. That difference is key to helping determine that (C) is correct.

In answer to your broader question about two answers weakening the argument, Law Services really tries to avoid that from happening, but, they inoculate themselves by usually including the word "most" before the "weakens." In the case where you think that two answers do weaken the argument, you would then choose the one that weakened it more.

Please let me know if that helps. Thanks!
 niketown3000
  • Posts: 16
  • Joined: Jan 13, 2012
|
#3715
You guys are wonderful, and this forum is great! Thank you
 Tuothekhazar
  • Posts: 20
  • Joined: May 28, 2020
|
#77385
Based on the argument, apparently it attempts to justify the causation based on the correlation.

If that's the case, would not answer D also weaken the argument by stating that there is the other alternative cause of " less faith " to have a group of patients attending support group to " not " live longer ?

If it is true most of whom attending weekly supporting group have less faith in dealing with disease T, then it must be true that there is other possible causes to make them " not " live longer than the other group. Perhaps group offering supports, however, due to faith lost, undesired to fight disease T no more, be negatively impacted by their own psychological state. Which perfectly weaken the argument, right ?

I can see both A and C could weaken the argument in a sense that they both raise the other correlations that counter the original one. However, if C is correct answer, would not we must to assume that there is no any other exterior force to have patients from the weekly supporting group to live longer ? Also, what if within 43 patients of attending the weekly support group from the first date whom participated in the study " are " 2 years older than the other group of patients without attending the group ? Based on C, it does not explicitly indicate the term of " 2 years longer of life lived " happened within that 10 years segment. Which is to say, if answer C clearly stated that within 10 years, the average life span of whom attending supporting group is 2 years more than the other group, and both the negative and positive impact derived from standard deviation of the age of both group be discerned. Then C might be regarded as " potentially correct answer "

In terms of answer A, it attempts to equate the characteristic ( live longer ) of few samples to it of the whole group, and the attempt serves the best reason to be eliminated in the first place.

Perhaps I am absolutely wrong within some of my reasoning process; however, please help me out to improve.
 Jeremy Press
PowerScore Staff
  • PowerScore Staff
  • Posts: 1000
  • Joined: Jun 12, 2017
|
#77470
Hi Tuo,

Before getting into the answer choices, a reminder of what we're looking for in those answers. We want an answer that will weaken the conclusion that "support group meetings do not help patients with disease T live longer." That means we want an answer giving some support to the opposite: that support group meetings DO help patients with disease T live longer.

If anything, answer choice D might strengthen the argument. If the physicians in answer choice D are correct (big if!), then support group meetings give patients less faith in their treatments, and it's possible the patients' lack of faith makes their treatment less effective, so they don't live longer. This is not actually an alternate cause style of answer, because the answer makes clear there is a connection between the support group meetings and the lack of help for patients--they aren't helping specifically because of their psychological impact. That doesn't weaken the idea that meetings don't help. It just shows HOW (or why) those meetings don't help.

But it's actually better to identify a couple big problems with answer choice D: just because "some" physicians "argue" something doesn't mean those physicians are right about what they argue! So answer choice D already isn't very powerful. Furthermore, I really can't say for sure (nothing in the stimulus or answer choice D authorizes this assumption) that less faith in treatment definitely means that treatment is less (or more) effective. So answer choice D's effect on the argument is actually indeterminate.

You're being a little too harsh on answer choice C, though. Remember, it's not the job of the weaken answer to destroy the argument, just to make it somewhat less likely to be true. If the support group patients lived, on average, two years longer, that's good evidence that support group meetings are correlated with longer life. Is it proof of causation? Of course not. But if I'm the person making the argument, that fact (that correlation) is definitely NOT something I want to be true. It gets closer to the idea that support group meetings do actually help (it's something that would be true if support group meetings did help). That's all we need from a weaken answer.

Answer choice A is okay as far as it goes, but it's only about 2 people, whereas answer choice C is about an average over all 43 people. Much more powerful from a data perspective, thus answer choice C is the strongest weaken!

Jeremy
 ltowns1
  • Posts: 60
  • Joined: May 16, 2017
|
#96259
I have a small difference in the way I saw (A) and I was wondering if anyone could give insight into whether I’m correct.? I got this question right, but to me, (A) is wrong because of who it’s discussing. I agree it weakens slightly, but to me a bigger problem is it doesn’t really address the people who died. Seems like the argument wants you to acknowledge that for the people who did die, there was no benefit to group therapy. (C) directly challenges that idea by saying that there could’ve been a benefit to group therapy. It potentially helped them live longer than they would otherwise.
User avatar
 atierney
PowerScore Staff
  • PowerScore Staff
  • Posts: 215
  • Joined: Jul 06, 2021
|
#96957
I would agree with your assessment of both answer choices and note that answer choice A adds to the argument somewhat by implying that the survivors (of 10 years) didn't all come from the group attending the meetings. But essentially you are correct, C weakens the argument, and A doesn't at all.
 ltowns1
  • Posts: 60
  • Joined: May 16, 2017
|
#96994
Thanks for the reply!

Get the most out of your LSAT Prep Plus subscription.

Analyze and track your performance with our Testing and Analytics Package.